at 2 p.m., two cars leave eagle river, wi, one headed east and one headed west. if the car traveling east averages 55 mph and the car traveling west averages 40 mph, when will the cars be 237.5 miles apart?

Answers

Answer 1

After 2.5 hours cars be 237.5 miles apart.

What is distance?

The measurement of distance between two objects or points can be quantitative or occasionally qualitative. Distance in physics or common usage can refer to a physical length or an estimation based on other factors.

First to write equation for each car

[tex]d_1=r_1t_1 (east bound)\\d_2 = r_2t_2 (west bound)[/tex]

They both start at the same time, and I will stop them both when they are 165 mi apart.

So, [tex]t_1=t_2[/tex] call just t.

[tex]d_1 = r_1t\\d_2=r_2t[/tex]

[tex]t = \frac{d_1}{r_1} \\t= \frac{d_2}{r_2}[/tex]

[tex]\frac{d_1}{r_1} = \frac{d_2}{r_2} \\d_1r_2 = d_2r_1[/tex]            ..(1)

We also know that

[tex]d_1 + d_2 = 237.5[/tex]

[tex]d_2 = 237.5 - d_1[/tex]

[tex]r_1 = 55, r_2 = 40[/tex]

So, equation (1) becomes,

[tex]40d_1 = 55d_2[/tex]

Plug the value of [tex]d_2[/tex].

[tex]40d_1 = 55(237.5 - d_1)\\40d_1 = 13,062.5 - 55d_1\\40d_1 + 55d_1 = 13062.5\\95d_1 = 13062.5\\d_1 = 137.5[/tex]

[tex]d_1 = 137.5 mi[/tex]

Now,

[tex]t = \frac{d_1}{r_1} = \frac{137.5}{55} = 2.5[/tex]

Hence, after 2.5 hours cars be 237.5 miles apart.

To know more about distance, click on the link

https://brainly.com/question/26046491

#SPJ1


Related Questions

Using composition of functions, determine if the two functions are inverses of each other. , x ≥ 0 , x ≥ 2 A. No, because the functions contain different operations. B. No, because the composition does not result in an answer of x. C. Yes, because F(x) is equal to –G(x). D. Yes, because the composition results in an answer of x for x ≥ 2.

Answers

The correct option regarding whether the two functions are inverse functions is given as follows:

B. No, because the composition does not result in an answer of x.

How to verify if the two functions are inverses of each other?

Two functions are inverses of each other if the composition of these two function has a result of x.

From the image given at the end of the answer, the two functions in this problem are defined as follows:

[tex]F(x) = \sqrt{x} + 4, x \geq 0[/tex]G(x) = x² - 4, x ≥ 2.

Their composition is given as follows:

[tex](G \circ F)(x) = G(\sqrt{x} + 4)[/tex]

[tex](G \circ F)(x) = (\sqrt{x} + 4)^2 - 4[/tex]

[tex](G \circ F)(x) = x + 8\sqrt{x} + 16 - 4[/tex]

[tex](G \circ F)(x) = x + 8\sqrt{x} + 12[/tex]


The composition does not result in x, hence the two functions are not inverses and the correct statement is given by statement B.

Missing Information

The problem is given by the image shown at the end of the answer.

More can be learned about inverse functions at https://brainly.com/question/3831584

#SPJ1

y = -2x + 1
4x + y = 9

Solve the system of equations without graphing. Show all the math steps needed to find the solution to this system.

Answers

Answer:

x =4 & y = -7

Step-by-step explanation:

this is the right answer

Find the sum of 5.21 × 1019 and 3.149 × 1017.

A. 8.359 × 1019
b. 5.24149 × 1019
c. 5.24149 × 1036
d. 8.359 × 1036

Answers

Answer:

B

Step-by-step explanation:

5.21 x 10^19 + 3.149 x 10^17

10^17(5.21 x 10^2 + 3.149)

10^17(521 + 3.149)

10^17(524.149)

5.24149 x 10^17 x 10^2

5.24149 x 10^(17+2)

5.24149 x 10^19

Answer: b. 5.24149 × 1019

Step-by-step explanation: I took the test

PLEASE HELP URGENT WITH 4 AND 6

Answers

The solution is

a) The compound interest for the second period is $ 2.01 and the amount after the second period is $ 404.01

b) The compound interest for the first period is $ 114.125 and the amount is $ 18,374.125

The compound interest for the second period is $ 114.83828125 and the amount after second period is $ 18,488.96

What is Compound Interest?

Compound interest is interest based on the initial principle plus all prior periods' accumulated interest. The power of compound interest is the ability to generate "interest on interest." Interest can be added at any time, from continuously to daily to annually.

The formula for calculating Compound Interest is

A = P ( 1 + r/n )ⁿᵇ

where A = Final Amount

P = Principal

r = rate of interest

n = number of times interest is applied

b = number of time periods elapsed

Given data ,

a)

Let the principal be P = amount after first period =  $ 402

Now , the rate of interest r = 6 %

The compound interest is calculated by compounding monthly

Now , the interest for the second period = ( PRT / 100 x 12 )

Substituting the values in the equation , we get

The interest for the second period = ( 402 x 6 x 1 ) / 1200

The interest for the second period = $ 2.01

The amount after the second period = amount after first period +  interest for the second period

The amount after the second period = 402 + 2.01

The amount after the second period = $ 404.01

b)

Let the principal be P = $ 18260

The rate of interest r = 2.5 %

The compound interest is calculated by compounding quarterly

Now , the interest for the first period = ( PRT / 4 x 100 )

Substituting the values in the equation , we get

The interest for the first period = ( 18260 x 2.5 ) / 400

The interest for the first period = 45650 / 400

The interest for the first period = $ 114.125

The amount for the first period = 18260 + 114.125

The amount for the first period = $ 18,374.125

Now , the principal for the second period = interest + amount of first period

The principal for the second period = $ 18,374.125

The rate of interest = 2.5 %

The interest for the second period = ( PRT / 4 x 100 )

Substituting the values in the equation , we get

The interest for the second period = ( 18374.125 x 2.5 ) / 400

The interest for the second period = 45935.3125 / 400

The interest for the second period = $ 114.83828125

The amount for the second period = 18,374.125+ 114.84

The amount for the second period = $ 18,488.96

Hence , the amount and interest is calculated

To learn more about compound interest click :

https://brainly.com/question/21270833

#SPJ1

1. Given a mean of 165 and a standard deviation of 4, use the empirical rule to answer the following questions: a. What percent is less than 161? b. What percent is more than 173? c. What percent is between 157 and 169? d. What is the 84 th percentile?

Answers

Using a mean and standard deviation as inputs, the following formula can be used to get the percentile of a normal distribution:

Percentile Value is equal to z +.

What is standard deviation?

The standard deviation is a statistic that expresses how much variance or dispersion there is in a group of numbers.

While a high standard deviation suggests that the values are dispersed over a wider range, a low standard deviation suggests that the values tend to be close to the mean (also known as the anticipated value) of the collection.

The lower case Greek letter (sigma), for the population standard deviation, or the Latin symbol s, for the sample standard deviation, are most frequently used in mathematical texts and equations to indicate standard deviation.

Standard deviation may be written as SD.

A random variable, sample, statistical population, data set, or probability distribution's standard deviation is equal to the square root of its variance.

Though in algebra, it is simpler

According to our question-

a. 157 is less then 161

b. n>173, n be a real number.

Hence, Using a mean and standard deviation as inputs, the following formula can be used to get the percentile of a normal distribution: Percentile Value is equal to z +.

Learn more about standard deviation click here:

https://brainly.com/question/475676

#SPJ1

what is the product of each equation?
[tex]\left(3a^{2}b^{4}\right)\left(-8ab^{3}\right)[/tex]
[tex]\left(3a^{2}b^{7}\right)\left(5a^{3}b^{8}\right)[/tex]
[tex]\left(-2d^{2}+s\right)\left(5d^{2}-6s\right)[/tex]
[tex]\left(3x-6\right)\left(2x^{2}-7x+1\right)[/tex])
[tex]\left(7x^{2}y^{3}\right)\left(3x^{5}y^{8}\right)[/tex]
[tex]\left(y^{2}+3y+7\right)\left(8y^{2}+y+1\right)[/tex]
[tex]\left(4s+2\right)\left(5s^{2}+10s+3\right)[/tex]

Answers

The solution to the products of exponents is as follows;

1) -24a³b⁷

2) 15a⁵b¹⁵

3) -10d⁴ + 17sd² - 6s²

4) 6x³ - 33x² + 45x - 6

5) 21x⁷y¹¹

6) 8y⁴ + 25y³ + 60y² + 10y + 7

7) 20s³ + 50s² + 32s + 6

How to find product of exponents?

The Product of Powers Property of exponents states that when we multiply two powers with the same base, we add the exponents.

1) (3a²b⁴) * (-8ab³)

= -24a³b⁷

2) (3a²b⁷) * (5a³b⁸)

= 15a⁵b¹⁵

3) (-2d² + s)(5d² - 6s)

Expanding the bracket to get;

(-10d⁴ + 5sd² + 12sd² - 6s²)

= -10d⁴ + 17sd² - 6s²

4) (3x - 6) * (2x² - 7x + 1)

= 6x³ - 33x² + 45x - 6

5) (7x²y³) * (3x⁵y⁸)

= 21x⁷y¹¹

6) (y² + 3y + 7) * (8y² + y + 1)

= 8y⁴ + 25y³ + 60y² + 10y + 7

7) (4s + 2) * (5s² + 10s + 3)

= 20s³ + 50s² + 32s + 6

Read more about product of Exponents at; https://brainly.com/question/847241

#SPJ1

what is the equation for a cosecant function with vertical asymptotes found at x equals pi over 4 plus pi over 4 times n comma such that n is an integer?

Answers

The answer is that the the equation for a cosecant function with vertical asymptotes found at x equals pi over 4 plus pi over 4 times n comma such that n is an integer is 2 coscec 4x.

Cosecant is one of the trigonometric ratios . It is calculated as the ratio of the hypotenuse to the perpendicular of a right triangle. It is also calculated as the reciprocal of the sine function.

Now, here if the vertical asymptotes is x = pi/4 + pi/4 (n).

This is not defined when n=0.

so, when x= pi/4,

sin 4x = sin x = 0

which means w =0 .

and we get the equation for cosecant function  to be f(x)= 2 cosec 4x since cosec x = 1/ sin x.

To learn more about trigonometry, visit link -https://brainly.com/question/29002217

#SPJ4

PLEASE EXPLAIN WHY YOU GOT YOUR ANSWER PLEASEE!!!!
When Skip was a puppy, he weighed 5.6 lbs. When he visited the vet, he weighed 13.3 lbs. How much weight did Skip gain between the time he was a puppy and his vet visit?

Answers

Answer:

7.7

Step-by-step explanation:

I got this by subtracting 13.3 - 5.6

Let f(x) = e ^ (6x) and g(x) = 8in(x) Find and simplify g(f(- 2)) -96 -48 48 C B A

Answers

Answer: g(f(-2))=-96

Step-by-step explanation:

[tex]Given: \ f(x)=e^{6x}\ \ \ \ g(x)=8ln(x)\ \ \ \ g(f(-2))=?\\\\f(-2)=e^{6*(-2)}\\\\f(-2)=e^{-12}\\\\Hence,\\\\g(f(-2))=8ln(e^{-12})\\\\g(f(-2))=8*(-12)\\\\g(f(-2))=-96[/tex]

Answer:

-96

Step-by-step explanation:

Given:

[tex]\begin{cases}f(x)=e^{6x}\\ g(x)=8 \ln (x) \end{cases}[/tex]

To find g[f(-2)], substitute x = -2 into the function f(x):

[tex]\implies f(-2)=e^{6 \times -2}=e^{-12}[/tex]

Then substitute the function f(-2) in place of the x in function g(x):

[tex]\implies g[f(-2)]=8 \ln \left(e^{-12}\right)[/tex]

[tex]\textsf{Apply the power law}: \quad \ln x^n=n \ln x[/tex]

[tex]\begin{aligned}\implies g[f(-2)]&=-12\cdot 8 \ln \left(e\right)\\&=-96 \ln \left(e\right)\end{aligned}[/tex]

Apply the log law:  ln(e) = 1

[tex]\begin{aligned}\implies g[f(-2)]&=-96 \ln \left(e\right)\\&=-96(1)\\&=-96\end{aligned}[/tex]

---------------------------------------------------------------------

As one calculation:

[tex]\begin{aligned}g[f(-2)]&=8 \ln \left(e^{6 \times -2}\right)\\& = 8 \ln \left(e^{-12}\right)\\& = -12 \cdot 8 \ln \left(e\right)\\& = -96(1)\\& = -96\end{aligned}[/tex]

Select the correct answer. Solve the following inequality. 21 ≤ -3(x − 4) < 30 A. -3 ≤ x < 6 B. -10 < x ≤ -3 C. 28 ≤ x < 37 D. -6 < x ≤ -3

Answers

Answer:

D. -6 < x ≤ -3

Step-by-step explanation:

21 ≤ -3(x − 4) < 30

21/3 ≤ -3(x − 4)/3 < 30/3

7 ≤ -(x − 4) < 10

7 ≤ -x - (-4) < 10

7 ≤ -x + 4 < 10

7 - 4 ≤ -x + 4 - 4 < 10 - 4

3 ≤ -x < 6

(3 ≤ -x < 6)/(-1)

-3 >= x > -6   ==> when dividing by a negative number, switch the inequality          

                           sign

-6 < x ≤ -3 ==> D

if the odds on a particular football team winning a bowl game are 2:3 that means the percent chance that the team will win is:

Answers

There will be 40% chance that the team will win.

Odds, are given as (chances for success) : (chances against success) or vice versa.

If odds are stated as an A to B chance of winning then the probability of winning is given as P(winning) = A / (A + B) while the probability of losing is given as P(Losing) = B / (A + B).

For 2 to 3 odds for winning:

P(winning) = 2/2+3

                 = 2/5

                 = 0.4

⇒ The percent chance of winning is 40.

Therefore, 40% is the required answer.

Learn more in depth about the probability at https://brainly.com/question/24756209

#SPJ4

0.55 / 2what is the probability that neither a nor b will happen? (round your answer to 2 decimal places.)

Answers

(a) The probability of either A or B occurring is 0.81.

(b) The probability that neither A nor B will happen is 0.19.

In the given question,

P(A)=0.39 P(A) = 0.39 and P(B)=0.42. P(B) = 0.42

(a) We have to find the probability of either A or B occurring.

P(A)=0.39

P(B)=0.42.

Since, events A and B are mutually exclusive

P(A and B) = 0

P(A or B) = P(A) + P(B) - P(A and B)

P(A or B) = 0.39 + 0.42 - 0

P(A or B) = 0.81

Probability of either A or B occurring is = 0.81

(b) Now we have to find the probability that neither A nor B will happen.

P(A or B) = 0.81

Probability that neither A nor B will happen = 1 - P(A or B)

Probability that neither A nor B will happen = 1 - 0.81

Probability that neither A nor B will happen = 0.19

To learn more about probability link is here

brainly.com/question/11234923

#SPJ4

The right question is:

The events A and B are mutually exclusive. Suppose P(A)=0.39 P(A) = 0.39 and P(B)=0.42. P(B) = 0.42.

(a) What is the probability of either A or B occurring? (Round your answer to 2 decimal places.)

(b) What is the probability that neither A nor B will happen? (Round your answer to 2 decimal places.)

Trent and his family are heading to Perfect Putt Mini-Golf. They plan to purchase the group package. With the package, the cost per person is $3 less than the normal cost for an individual. There are 6 people in Trent's family. His mom called ahead and found that the total cost for the family will be $30.
Which equation can you use to find the normal cost, x, for an individual?

3x-6=30
6(x-3)=30
6x-3=30
3(x-6)=30

Answers

ANSWER:

6(x-3)=30

EXPLANATION:

I did the math and I’m really good at math.

PLS HELP W/ THESE LAST FEW QUESTIONS IM STRUGGLING THANK UU!!! (47 PNTS)
1) a
Obtuse and scalene
b
Isosceles and scalene
c
Acute and right
d
Right and scalene
e
Right and equilateral


2)Find the measure of the missing angle.

3)Find the measure of the missing angle.

6) t= ; ∠X

8) x=

10) m∠R = ; SR

Answers

Answer:

1. Scalene Triangle

2. 135 degrees

3. 80 degrees

I don't have much time for the rest, i'm sryyyyy

I hope i could helppp :3

rotation 90 degrees counterclockwise about the origin

Answers

As per the sign system of the Cartesian plane coordinates of A' will be (x,-y).

What is the cartesian plane?

The cartesian plane is defined in mathematics as a two-dimensional coordinate plane formed by the intersection of the x- and y-axes. The origin is the point where the x- and y-axes intersect perpendicularly.

What are the quadrants?

Quadrant one (QI) is the top right fourth of the coordinate plane, where all coordinates are positive. Quadrant two (QII) is the coordinate plane's top left fourth. Quadrant three (QIII) is the fourth from the bottom left. Quadrant four (QIV) is the fourth from the bottom right.

Given:

The direction of rotation = clockwise, (from the first quadrant to forth quadrant then third quadrant, and so on)

Angle of rotation = 90° about the origin

According to the question, let's take our start point as A(x, y)

So, A( x, y) will lie in the first quadrant.

After 90° rotation about the origin A' will lies in fourth quadrant.

As per the sign system of the Cartesian plane coordinates of A' will be (x,-y).

To know more about the cartesian plane visit:

https://brainly.com/question/28574364

#SPJ1

Which of the following expressions are equivalent to 16/24? Select all that apply.

Answers

After evaluation, we can conclude that the expression (E) 8/2 * 2/12 is equivalent to the given expresion 16/24.

What are expressions?

Mathematical expressions consist of at least two numbers or variables, at least one arithmetic operation, and a statement.

It's possible to multiply, divide, add, or subtract with this mathematical operation.

You must substitute a number for each variable and carry out the arithmetic operations in order to evaluate an algebraic expression.

So, we have the expression:

16/24

We need to find the equivalent expression from the options as follows:

So, let's take option (E) as it seems the answer and evaluation is:

8/2 * 2/12 (Be LHS)

Now, multiply:

8/2 * 2/12 = 16/24

16/24 = 16/24

Therefore, after evaluation, we can conclude that the expression (E) 8/2 * 2/12 is equivalent to the given expression 16/24.

Know more about expressions here:

https://brainly.com/question/28934492

#SPJ1

can you please help me ASAP

Answers

Answer:

D. 7

Step-by-step explanation:

[tex] \frac{ \sqrt{196} }{\sqrt{4} } \\ [/tex]

[tex] \frac{14}{2} [/tex]

=7

Which number line shows the solutions to n > -2?
++++
+
-6-5-4-3-2-1 0 1 2 3 4 5 6
←++++
H
-6-5-4-3 -2 -1 0 1 2 3 4 5 6
+++
++++++
-6-5-4-3-2-1 0 1 2 3 4 5 6
+++++
-6-5-4-3-2-1 0 1 2
3 14 5 6
Done -
2

Answers

2888483829291929293983 298383

Solve for y. Find m∠ C and m∠ D. Show all work.

Answers

The value of y is 10. The measure m∠ C and m∠ D are 40° and 76° respectively.

What is external angle?

The angle created when a transversal cuts one of two lines and is located outside of the line. It describes the angle between a side of a polygon and an expanded adjacent side.

The measure of an external angle of a triangle is equal to the sum of opposite interior angles.

∠DEF is an exterior angle. The opposite interior angles of ∠DEF are m∠ C and m∠ D

Therefore,

m∠ C + m∠ D = ∠DEF

4y° + (7y + 6)° = 116°

Add like terms

11y + 6 = 116

Subtract 6 from both sides:

11y = 110

Divide both sides by 11:

y = 10

Putting y = 10 in  m∠ C = 4y° and m∠ D =  (7y + 6)°

m∠ C = 40°

m∠ D = 76°

To learn more about interior angle, click on below link:

https://brainly.com/question/20333276

#SPJ1

-8x - 5(-5x - 1) - 7 = 3(x − 3)

Answers

Answer:

-8x+25x+5-7=3x-9

or ,17x -2=-9+2

or, -14x=-7

or, x=-7/-14

x=1/2

Answer:

-8x +25x +5-7=3x-9

17x-2 = 3x-9

17x-3x=9+2

14x =11

x= 14/11

The library sponsors a chess club for members of all ages and skill levels. Currently, the ages of the members are 7, 9, 10, 11, 13, 14, 15, 16, 18, 19, 20, 21, and 22. The librarian uses a histogram to track the number of members in different age groups. For this situation, which is the appropriate way to label the age intervals on the x-axis?

Answers

Answer:

B. 7−10; 11−14; 15−18; 19−22

Step-by-step explanation:

PLEASE HELP ASAP IM GONNA FAIL:(

Answers

Answer:

The first answer on the bottom goes with the picture on top.

The second answer on the bottom goes with the table on the bottom.

Step-by-step explanation:

It is just asking you to list the x values and the y value.  The x values are the domains and the y values are the ranges.  That's it.

Name the property of real numbers illustrated by the equation. -2(x 4)=-2x-8

Answers

Distributive property of real numbers illustrated by the equation. -2(x 4)=-2x-8

The equation -2(x 4)=-2x-8 is an example of the distributive property of real numbers. This property states that when a number is multiplied by a group of numbers that are being added together, the number can be distributed to each of the terms in the group. To rewrite this equation as -2x + (-2)(-4) = -2x - 8, the number -2 is allocated to each of the terms in the group (x and -4) in this equation.Simplifying further, we get -2x - 8 = -2x - 8.

-2(x 4) = -2x - 8

Distribute -2 to each term in the group

-2x + (-2)(-4) = -2x - 8

Simplify

-2x - 8 = -2x – 8

Learn more about real number here

https://brainly.com/question/10547079

#SPJ4

For this lab, you will find the area of an irregularly shaped room with the shape as shown above.

Ask the user to enter the values for sides A, B, C, D, and E and print out the total room area.

Remember the formula for finding the area of a rectangle is length * width and the area of a right triangle is 0.5 * the base * height.

Please note the final area should be in decimal format.

Sample Run
Enter side A: 11
Enter side B: 2
Enter side C: 4
Enter side D: 7
Enter side E: 1
Sample Output
Room Area: 53.5

Answers

The python program code is written to find the area of the shape.

What is meant by area?

The size of a region on a planar or curved surface is expressed mathematically as area. The term "surface area" refers to the area of an open surface or the perimeter of a three-dimensional object, whereas "plane area" refers to the area of a shape or planar lamina.

The program is given as:

print('Please, enter sides lengths:')

# Getting sides lengths

a, b, c, d, e = map(float, (input('A = '), input('B = '), input('C = '), input('D = '), input('E = ')))

# Finding a rectangle area

rect_area = a * b

# Finding a triangle area

triang_height = (c**2 - (a / 2)**2)**0.5

triang_area = 0.5 * a * triang_height

# Sum area

area = rect_area + triang_area

print('Total room area: %f' % area)

To know more about rectangles, visit:

https://brainly.com/question/29085356

#SPJ1

you are skiing down a mountain with a vertical height of 2900 feet. the distance from the top of the mountain to the base is 5800 feet. to the nearest degree, what is the angle elevation from the base to the top of the mountain?

Answers

Answer:

  30°

Step-by-step explanation:

You want the angle of elevation to a point 2900 feet up that is 5800 feet distant on a straight line.

Sine

The sine of the angle is the ratio of the height to the straight-line distance.

  sin(θ) = h/d = 2900/5800 = 1/2

  θ = arcsin(1/2) = 30°

The angle of elevation is 30° from the base to the top.

Answer: When doing a problem like this I always draw an example so I can see it. If you do that you see a right triangle with the hypothenuse, 2500 feet, is the distance from the top to the base. The opposite side is 1250 feet. Then remember  sine= o/h, (opposite side over hypothenuse). So divide 1250/2500 and you get .5  Now take your calculator and use "inv" or sin-1 button to get the answer of the angle. x = 30

sinθ 1250/2500 = 1/2; θ = sin-1(1/2) = 30°

Step-by-step explanation:

If using the method of completing the square to solve the quadratic equation x 2 + 2 x + 16 = 0

Answers

Answer:

Step-by-step explanation:

[tex]x^{2} +2x+16=0\\x^{2} +2x+1-1+16=0\\(x^{2} +2x+1)-1+16=0\\(x+1)^{2} +15=0\\(x+1)^{2}=-15[/tex]

The solution does not exist because all numbers squared are greater than zero.

What number does B stand for
What number does C stand for

Answers

Answer:

B=20

C = 40  

These are the answers

49: 1 of 1 question 2 with 1 blank 97: 1 of 1 question 3 with 1 blank 113: 1 of 1 question 4 with 1 blank 632: 1 of 1 question 5 with 1 blank 1.781: 1 of 1 question 6 with 1 blank 3.558: 1 of 1 question 7 with 1 blank 1.006.015: 1 of 1 question 8 with 1 blank 67.224.370: 1 of 1 3

Answers

Spanish is a Romance language of the Indo-European language family that emerged from the Iberian peninsula's colloquial Latin. It is now a worldwide language with around 500 million native speakers, primarily in the Americas and Spain. Twenty nations have Spanish as their official language.

These numbers in Spanish are:

1. 49: cuarenta y nueve

2. 97: noventa y siete

3. 113: ciento trece

4. 632: seiscientos treinta y dos

5. 1.781: mil setecientos ochenta y uno

6. 3.558: tres mil quinientos cincuenta y ocho

7. 1.006.015: un millon seis mil quince

8. 67.224.370: sesenta y siete millones doscientos veinte y cuatro mil trescientos setenta

To know more about Spanish numbers refer to:

brainly.com/question/18035113

#SPJ4

Answer:

49: cuarenta y nueve

2. 97: noventa y siete

3. 113: ciento trece

4. 632: seiscientos treinta y dos

5. 1.781: mil setecientos ochenta y uno

6. 3.558: tres mil quinientos cincuenta y ocho

7. 1.006.015: un millon seis mil quince

8. 67.224.370: sesenta y siete millones doscientos veinte y cuatro mil trescientos setenta

Step-by-step explanation:

HELP ME PLEASE!!
I HAVE NO IDEA WHAT IM DOING!!
A. 36
B. 54
C. 72
D. 18

Answers

Answer:

72

Step-by-step explanation:

We are told that line AC divides angle BAE in half or "bisects" it. We are also told that line AD divides angle CAE in half as well. Lastly we are giving the value of angle DAE which is 36.

Since line DA split angle CAE in half and we have half of the value, that means that the measure of angle CAE is 36 times 2 which is 72.

72 happens to be our answer for angle BAC because angle BAC is half of the largest angle BAE, which was divided in two by line AC.

Hope this helps!

This graph shows all the key characteristics of a particular polynomial function. Drag tiles to complete the function rule that could be represented by the graph

Answers

The function rule that could be represented by the graph is given as follows:

y = 2x³ + x - 3.

How to define the function?

The first step in defining the function is looking at it's y-intercept, which is the value of y when the graph of the function crosses the y-axis.

This value is of -3, hence the function can be defined as follows:

f(x) = ax^n + x - 3.

Then we look at the behavior of the function. It has an inflection point, as it is concave down and then it becomes concave up, hence:

It's second derivative is of the first degree.It's first derivative is of the second degree.The function is of the third degree, hence n = 3.

Thus:

f(x) = ax³ + x - 3.

Then we look at the x-intercept, meaning that when x = 1, y = 0, this the leading coefficient is obtained as follows:

0 = a + 1 - 3

a = 2.

Thus the function is defined as follows:

y = 2x³ + x - 3.

More can be learned about functions at https://brainly.com/question/24808124

#SPJ1

Other Questions
You drop a 2.00 kg book to a friend who stands on the ground at distance D = 10.0 m below. If your friend's outstretched hands are at distance d = 1.50 m above the ground (Fig. 8-28), (a) how much work Wg does the gravitational force do on the book as it drops to her hands? (b) What is the change U in the gravitational potential energy of the bookEarth system during the drop? If the gravitational potential energy U of that system is taken to be zero at ground level, what is U(c) when the book is released and (d) when it reaches her hands? Now take U to be 100 J at ground level and again find (e) Wg, (f) U, (g) U at the release point, and (h) U at her hands.The aswers are as follows:a) 167J, b) -167J, c) 196J, d)29J, e)167J, f)-167J g)296J, and h) 129J how would you respond to a naysayer that says store business and concessions business will go down if we give out free hats and free food as part of group sales packages? What is the name of fossil 126 A 59-kgkg person riding a bike puts all her weight on each pedal when climbing a hill. The pedals rotate in a circle of radius 18 cmcm . What is the maximum torque she exerts? according to a united nations summary of findings on delinquency, each of the following is accurate except: watching your audience for nonverbal cues that indicate their level of interest in your topic is also known as One number is twelve less than another. If their sum is increased by seven, the result is 83please help me In a box of chocolates, of the chocolates are white chocolate, are milkchocolate, and the rest are dark chocolate.What fraction of the chocolates are dark chocolate? The _____ is a joint venture between the government and colleges/universities run by the SBA.O Chamber of CommerceO SBAO SCOREO SBDC The third movement in most Classical symphonies is generally _________. Part F: Order the integers from least to greatest. 12, -6, 20, -47, -11 There were two candidate A and B conteting for the municipal corporation election in a town. The total number of voter in the town were 80,000. If 90% of the voter had voted and 60% of vote polled were cat in favour of A, find the number of vote received by B Please hurry and help!Graph the following linear equation using the intercepts.\displaystyle -10 x-12 y=-6010x12y=60 After meeting with Ramie for several sessions, Dr. Morris discusses her prognosis. Dr. Morris is most likely talking aboutthe future course and probable outcome of Ramie's disorder. genetically encoding luorosulfate-l-tyrosine to react with lysine, histidine, and tyrosine via sufex in proteins in vivo. the former presidential candidate who became a leading spokesman against the theory of evolution during the 1920s was which of the equations below can be used to solve for the temperature at which a reaction becomes spontaneous?Choose one: -(AG-AS) AH -(AG-AH) -(AG+AH) AS AG I AH - AS Amy and Bob have decided to borrow a portion of the purchase price of stocks from their broker. They will then have a ________ account.A) cashB) round lotC) margin D) debtorE) combination Write a linear function f with the values. Suppose the total monetary value of all final goods and services produced in a particular country during 2010 is $600 billion, and the total monetary value of final goods and services sold is $450 billion. We can conclude that ___. real GDP in 2010 is $450 billion nominal GDP in 2010 is $600 billion inventories in 2010 fell by $50 billion nominal GDP in 2010 is $500 billion